10
$\begingroup$

Here is a chess puzzle of a somewhat different flavour than my usual stuff. I hope you like it. :)

I had just witnessed a chess game with a rather interesting sequence of moves in the endgame. It started by one of the players giving a discovered check on the opposing king. Then, for four single moves in a row, each player moved a piece that blocked the check they were in and also gave check in return. Intrigued by this phenomenon, I hastened to take a picture of the resulting position to study it later, but unfortunately knocked almost all of the pieces off the board (clumsy me)! The only piece remaining was the black king, which can be seen in this sorry photograph I took of the situation:

The lonely bK on d1

At least he, somewhat miraculously, remained in his correct square during all this mess. I've been trying hard to reconstruct where all the other pieces were and how the sequence had played out, but so far have not been successful. All I remember is that no pieces were captured during this and that everything but the pieces giving the checks in it, were taken long before. Can you help me out here?

What was the move that started the sequence?


(If you don't care for the fluff, here are the straight facts again:

  • The last five single moves started with a discovered check, followed by four moves, in which a piece simultaneously blocked a check and gave a check of its own.

  • No piece was taken during this sequence. Also, the board contained only the kings and the pieces that gave a check in it.

  • The black king is on d1.)

Please feel free to have a go at this! Have fun :)


Note: The original puzzle was flawed (as kindly indicated by Albert Lang). I had failed to word the second constraint the right way. This new formulation now (hopefully) fixes the puzzle. :)


Edit: It may be time for a little hint on how to show uniqueness (without endless grinding of cases):

Hint 1:

Suppose some piece Y blocks a check given by piece X. What do you notice about the triangles that each of these pieces forms with the two kings? What does that imply for a sequence of these events, that is, a sequence of pieces $X_1,...,X_n$ such that each piece $X_{k+1}$ blocks a check from piece $X_k$? How long can such a chain possibly be?

$\endgroup$
4
  • 1
    $\begingroup$ 8/8/8/1N4rB/1K4R1/8/1Q3q2/3k4 w - - 0 1 and 3R1Q2/8/8/3B4/6N1/1b6/6K1/3kr3 w - - 0 1 both seem to satisfy the constraints. Am I missing something? $\endgroup$ Commented Aug 10, 2024 at 12:15
  • 1
    $\begingroup$ @Albert.Lang Thanks a lot for pointing that out! You were quite right, the puzzle as stated was unfortunately flawed. I now reworked the second constraint to fix the solution (hopefully) $\endgroup$ Commented Aug 10, 2024 at 13:14
  • $\begingroup$ Could you please clarify what "giving a check" means exactly? Specifically, in a discovered check, which piece is considered giving the check? The moving one? Or the discovered one? Or both? Or both only if it is actually double check? $\endgroup$ Commented Aug 11, 2024 at 6:31
  • $\begingroup$ Sure, my intended meaning is that a piece gives a check if and only if it threatens the square of the opposing king directly. (So indeed, the moving piece in a discovered check is generally not giving check itself.) $\endgroup$ Commented Aug 11, 2024 at 7:12

2 Answers 2

5
$\begingroup$

There is a graph related to the geometry of a chess board. The vertices are the squares of the board. We are ignoring for the moment the knights, and are joining two vertices if there is a line of attack (horizontal, vertical, or diagonal) between them, i.e. if a queen can (reciprocally) reach one square from an other. In this graph, between two connected vertices there is a notion of distance, the cartesian distance between the centers of the corresponding squares.

Note that only the last check may be a knight check.

Let $j, J$ be the two kings. One of them is the King on $d1$, but we do not know which one. The colors are a while separated by using lower and capital letters. We do not know so far which of them got checked first. At any rate, the story must have been as follows:

  • A piece $S$ moves, discovers the line of attack of the piece T, which gives a check to $j$.
  • A piece $u$ interposes between $T$ and $j$, and gives a check to $J$.
  • A piece $V$ interposes between $u$ and $J$, and gives a check to $j$.
  • A piece $w$ interposes between $V$ and $j$, and gives a check to $J$.
  • A piece $X$ interposes between $w$ and $J$, and gives a check to $j$.

It is given that S, the first piece moving, is either V or X, since each piece on the board gave a chess in the sequence of the five half-moves. All the time there were exactly $2+5=7$ pieces on the board.

In a picture using the lines of attack mentioned above, we have topologically a subgraph:

[T] (1)
 |
 |
[u] (2) 
 | \ 
 |  \ 
 |   \
 |    \
 |     \
 |      \
 |       \
 |        \
 |         \
 |          [V] (3)
 |         /  \
 |        /    \
 |       /      \
 |     /         \
 | (4)/           \
 | [w]----[X]-----[J]
 | /    .::(5)
[j]:::::

There is a final dotted line connecting $Xj$, which is either a connection in the graph, or not, and in the latter case it is a "night check".

  • Let $\Bbb H$ be the half-plane delimited by the line $jJ$ of the two kings in the plane of the board, which contains the first piece $T$ that gives the first check. It is clear by convexity, that all pieces live in their final position in $\Bbb H$. And even more in the "solid" triangle $\Delta_T$ with $T,j,J$ as vertices, which is the convex hull of these points.

  • Let $\Delta_u$ be the solid triangle (on the plane of the board) with vertices $u,j,J$. Then the "next" pieces $V,w,X$ live in their final position in this triangle, including boundaries.

  • We may further continue with smaller solid triangle regions to restrict the next points.

  • Which is the angle between the vectors $\overrightarrow{Tj}$ $\overrightarrow{uJ}$, measured by rotating $\overrightarrow{uj}$ (normed to length one) in the direction of $\overrightarrow{uJ}$ (normed to length one), seen as positive direction? It is a multiple of $45^\circ$ since two lines of attack in chess are so. It is easy to see that $135^\circ$ is excluded, because else in the triangle $\Delta ujJ$ the angle in $u$ is $135^\circ$, and no point $V$ can be furthermore chosen on $uJ$ to make an allowed angle $$ \widehat{uVj}< \widehat{uJj}< 180^\circ- \widehat{Juj}=180^\circ-135^\circ=45^\circ\ . $$

  • So only angles of $45^\circ$ and $90^\circ$ are allowed in the process.

  • Consider now the following configuration only:

      [u] (2) 
       | \ 
       |  \ 
       |   \
       |    \
       |     \
       |      \
       |       \
       |        \
       |         \
       |          [V] (3)
       |         /  \
       |        /    \
       |       /      \
       |      /        \
       |     /          \
       |    /           [J]
       |   /
       |  /
       | /
      [j]
    

    and in the triangle $\Delta ujJ$ the angle in $u$ is bigger than the angle in $V$ in $\Delta VjJ$.

  • With this argument all vectors $\overrightarrow{Tj}$ $\overrightarrow{uJ}$, $\overrightarrow{Vj}$, $\overrightarrow{wJ}$, moved parallely to have the same origin, say $T$, stay in the half-plane $\Bbb H$, including its boundary, and have (strictly) increasing consecutive angles (with alternating orientationa), which are multiples of $45^\circ$.

  • So it is not possible to have one more half-move, and in our situation

    • the angle from $\overrightarrow{uj}$ to $\overrightarrow{uJ}$ is the minimal possible angle $45^\circ$,
    • the angle from $\overrightarrow{Vj}$ to $\overrightarrow{VJ}$ is the minimal possible angle $90^\circ$,
    • the angle from $\overrightarrow{wj}$ to $\overrightarrow{wJ}$ is the minimal possible angle $135^\circ$.
    • the piece that interposes now and gives a check must be a knight, so a knight $S=X$ was the piece moved first, discovering the line of attack from $T$ to $j$.
  • If $j$ is the black king, then the first line of attack was not a diagonal, since a night on a light square (initially on this diagonal) cannot move twice and attack also a light square like d1. (Initially it was on a light square, which attacks only black squares. The sames happens in two moves.) If it was the vertical, then $T$ is a rook or a queen on the $d$-file, without restriction of generality it is $Rd8$. Then $u$ interposes and we must have a diagonal check, so it is a light square, as $d1$. In particular $u$ is also a diagonally working piece, without restriction it is a queen on one of the squares $d7$, $d5$, $d3$. Without restriction, we can reflect the position if needed w.r.t. the $d$-line so that the white king is on the king side. The last piece $w$ gives a check and a knight interposes with check. This is only possible with a knight on the king side on $f2$. (The other square $e3$
    is easily excluded.) The only possibility for $w$ is in the final position $Re2$ or $Qe2$. So the white king is on the second line, and there is only one light square beyond $f2$, which is $g2$. So $u$ is a $Qd5+$ (or $Bd5$), $V$ is the $Qf3+$. And the only final position that may occur is:

$\hspace{2cm}$puzzling SE :: 1st possibility

But how can a $N$ move starting from the $d$-file twice, reach $f2$ in two moves, and not interfere with other lines of attack? It must start on a black square. The path $d4-(?)-d2$ is not possible, the remained black squares are $d2,d6$, and $f2$ can be reached only through $e4$. Which interferes. No such solution. (We would have one with a $-1$-line, the first knight move would put it on $e(-1)$, from where $f2$ can be reached.

  • If $j$ is the black king, and the first line of attack is a horizontal, then we may assume that everything happens on the king side, and we deduce successively: A rook $T$ gives check from the first line, a bishop $u$ interposes, so the white king sits diagonally on its line of attack. A queen $V$ (or a bishop, but it is harder to come there as bishop, so in such situations we take the queen,) interposes and checks the $Kd1$, so it sits on a light square, so the white king is also on a light square. So the bishop $u$ is on a light square. Only $f1$ is available, but this leads to no solution. We would have a solution on a board with nine columns, when the final position is...

$\hspace{2cm}$puzzling SE :: 2nd position, rejected on the usual chess board

Which can be reached by $1.Ne1-c2+$ (from $Ri1$) $1\dots\ Qh2-h1+\ 2.Qd4-d3+ \ Qd2-e2+\ 3.Nc2-e3+$ - but we need that $i$-file.

  • So $j$ is the white king, and the first piece giving the check is a black piece, and there is a knight move that discovers the line of attack. The same parity reason forbids a diagonal as first line of attack. (The black knight and the white king would be on a square of same color, after two (or after and even number of) moves the knight attacks the opposite color.) We can realize a position with a first check on a horizontal line as follows:

$\hspace{2cm}$puzzling SE :: 3rd position, a solution

$$1\dots\ Nd4-b3+\ 2.Bh3-g4+\ Qe1-e2+\ 3.Qc3-d3+\ Nb3-d2+\ .$$ What about a vertical line? No, we would need a line below the black king, in fact even two lines for the first knight move, in order to realize

$\hspace{2cm}$puzzling SE :: 4th position, rejected on the usual chess board

To conclude, only a solution with a horizontal first check is possible, the third diagram shows a possible realization.

$\endgroup$
1
  • $\begingroup$ Very nice, thanks for putting in the effort! This is essentially the intended solution, with only one question remaining: How did you conclude that the first moving piece must have been the knight? Why could it not have been the second (uppercase) checking piece? (This does indeed follow from just determining the sequence of the angles, but I feel it warrants a more explicit reasoning.) Other than that, very nice work! :) $\endgroup$ Commented Aug 14, 2024 at 9:03
6
$\begingroup$

A possible solution appears to be:

Position 8/8/8/6Q1/2Kn3r/2R5/5q2/3k4 b - - 0 1 Black to move
8/8/8/6Q1/2Kn3r/2R5/5q2/3k4 b - - 0 1

The five moves being

1. .. Nb3+ 2. Qg4+ Qe2+ 3. Rd3+ Nd2+

But I have no idea how to show that this would be unique in any way. (Most of the starting positions clearly aren't. But the final positions might be.)

$\endgroup$
1
  • $\begingroup$ Looking good, this is the solution I had in mind (up to some trivial flexibility in the set of pieces). If you are interested in pursuing this further, I can assure you that there is a completely logical way to show uniqueness without having to split into annoying casework. (That way may probably best be found by ignoring the specific position of bK and pretending to play on an infinite grid. I might still wait on dropping more specific hints though for now.) Good work! :) $\endgroup$ Commented Aug 11, 2024 at 16:58

Your Answer

By clicking “Post Your Answer”, you agree to our terms of service and acknowledge you have read our privacy policy.

Start asking to get answers

Find the answer to your question by asking.

Ask question

Explore related questions

See similar questions with these tags.